User Avatar
ryanrt579
Joined
Apr 2025
Subscription
Free
User Avatar
ryanrt579
Tuesday, Jun 29 2021

Thanks so much for this comprehensive and insightful post, @ !

PrepTests ·
PT111.S4.Q1
User Avatar
ryanrt579
Saturday, Feb 27 2021

Another way that B can be deemed correct, is that the answer choice overreaches, with the statement "any mathematical understanding". The stimulus only discusses algebra and geometry, not mathematics within the entirely.

With Most Strongly Supported questions, answer choices that discuss aspects that are outside a narrowly defined stimulus, with options such as (i.e. ANY, all, etc.), are typically incorrect.

PrepTests ·
PT119.S2.Q10
User Avatar
ryanrt579
Sunday, Dec 27 2020

Another way to look at answer choice C) is, even if the job performance declines, it still might NOT be enough to have a disastrous impact; which is why its not necessary.

User Avatar
ryanrt579
Monday, Apr 26 2021

Hello rpt802,

Have you performed any timed tests? Or maybe just timed logical reasoning sections? For many people who do not score what they want to in LR, it can often boil down to not having a proper skipping strategy. Spending too much time on questions that you're properly going to get wrong, is not a good use of timing, as you can better spend it on the ones that you will get right. For instance, some people when they do the test, allow themselves to skip two or three (of the hardest) questions. This takes some of the pressure off and allows them to focus more on the ones they will get right.

I've found there's some merit in reviewing any old PTs that you have done, via blind review, to help yourself recognize some of the consistent patterns that are the trend with LR questions. As Fact_or_Feelings has mentioned, looking at the core curriculum is an excellent way to help shore up any issues you may be having.

Using the problem set generator to help you drill strengthen/weaken questions on a repetitive basis can also help you.

Remember, the LSAT can be difficult, but its still a standardized test, it has patterns that you can most-definitely learn and this in turn will help you improve your score. I myself am still going through the learning process and I can tell you, that things do get better, it can just take time.

Please feel free to message with anymore information - we're all here for you!

User Avatar

Sunday, Jul 26 2020

ryanrt579

PTF97.S1.Q13

Hello,

I'm confused as to how this answer is D. I'm torn between C and D. Is C wrong because if you fail the sufficient (don't have the Pterodactyl consume the red algae , the colouration could still be caused by other factors? Whereas D gives more indication that since it lived in an area that was abundant with red algae, that its more likely to have consumed it?

#help

PrepTests ·
PT111.S4.Q23
User Avatar
ryanrt579
Friday, Feb 26 2021

After reviewing this answer choice, I was able to get it right. A) is a comparision of relative manner, as JY says, person B who is younger, could have a higher "wise" base than person A who is older. However, with answer choice D, the comparison is absolute: the older something is (C) --> the more rings it has. C is older than D, so it must have more rings.

User Avatar
ryanrt579
Friday, Jun 25 2021

Firstly Alice777, congratulations on your excellent blind review score! That's great news that you're getting nearly perfect on a consistent basis!

One of the best ways (if not the best) to improve your LG score is simply to keep foolproofing the same game (as frenchfyre just provided a link explaining the process) until you have mastered it by getting a perfect score. The most likely explanation for the variance in scores between timed and BR is because that you aren't yet that proficient with the games that you can replicate your BR score when under timed pressure.

Keep at it! Eventually through sheer repetition and practicing under timed scenarios as well as BRing, you will eventually see your score variation dissipate, as I have.

All the best, and feel free to message me should you have any further questions.

Ryan

PrepTests ·
PT134.S3.Q9
User Avatar
ryanrt579
Saturday, Apr 24 2021

@amipp_93 and KevinSage have explanation explanations for anyone who is wondering between A and D. The latter has a term "no more than", this also means that A is less than or equal to B. This results in NOT an equal punishment between A and B, but possible scenarios where A's punishment might be equal, or less than B, so its completely wrong.

Thanks to both of you for shedding light on this!

PrepTests ·
PT131.S2.Q22
User Avatar
ryanrt579
Monday, Mar 22 2021

My explanation of answer choice A (I missed this during timed and BR):

You have a study, where you’re claiming that consumption of breakfast, ensures that workers become more productive. You point to an experiment, where workers onsite received free nutritious breakfasts before they started work. As a result, this group was more productive than the group that did not receive the free nutritious breakfasts. However, there’s a distinction between the premise of receiving the free nutritious breakfast, and consuming breakfast at all. Which is what answer choice A is getting at. In order to strengthen this, you need to ensure that the group that wasn’t productive did NOT consume breakfast on their own time (i.e. before their shift started). So the phrasing of it: “few workers in group B consumed nutritious breakfasts in the month during the study” {few did something = most did not (A—m-->-B). This would re-affirm the conclusion. For it was false, that would mean that most workers in group B did in fact consume breakfast and that would wreck the argument, that consuming breakfast leads to a more productive workforce.

PrepTests ·
PT131.S2.Q1
User Avatar
ryanrt579
Monday, Mar 22 2021

I feel that what messed me up on this question, is that with answer choice A, I brought outside knowledge into analyzing the answer choice. I figured that by eliminating the symptoms, that wouldn't address the notion that the disease is necessarily gone - it could still be active in the person. However, the other four answers are way too far off.

Note to others: if you eliminate an answer choice because you believe its wrong, but then find other answer choices that seem even worse off, then its a potential sign that the least-wrong answer choice, could be your correct answer.

PrepTests ·
PT130.S4.Q4
User Avatar
ryanrt579
Saturday, Mar 20 2021

Note: while answer choice C, states that some people can tell about a waltz, some, means “at least one”, and could theoretically be all, or most, so “most people could tell whether any given piece of music” is a possibility. Stating that most people can’t, means that there’s a possibility that at least a few can, so that’s the best match.

As JY has stated, this questions types are similar to parallel reasoning questions.

PrepTests ·
PT130.S4.Q2
User Avatar
ryanrt579
Saturday, Mar 20 2021

D is wrong because the conclusion within the stimulus is a "qualified" (limited) conclusion. It is stating that One never needs gloves or mittens to keep warm. It does not state that sweaters and/or thermal undershirts provide the best solution, just that they provide A solutoin. This is a relative comparison.

PrepTests ·
PT130.S1.Q25
User Avatar
ryanrt579
Friday, Mar 19 2021

Hi, I have a question about the phrase: "Handmade foundations are never found on wigs that do not contain human hair". I understand that the never is Group 4, but I always thought that the "not" would also be group 4. So wouldn't the "never" and the "not" be two cases of group 4s together?

#help

PrepTests ·
PT130.S1.Q20
User Avatar
ryanrt579
Thursday, Mar 18 2021

I now understand why B is wrong and E is correct. Because this is a “Must be True” and the question stem commands us to assume that everything above the stimulus is indeed true, that means that since Type B is more at risk of heart disease than Type A, then having some of type A shift over into type B would make their risk of heart disease increase. This makes E correct.

B is wrong because we know nothing about averages, also we have no idea what their starting numbers were.

PrepTests ·
PT117.S2.Q7
User Avatar
ryanrt579
Friday, Dec 18 2020

A and B could be considered premise-boosters; they already tell us something that we already know (listening to music at loud levels through stereo headphones is bad for your hearing). This wouldn't strengthen the author's conclusion. D on the other hand does, as if teenagers are aware of the risks of hearing loss and still decide to pursue the course, then they will simply disregard the headphones that have the shut-off feature and find ones that do not have it.

A tricky question due to the abstract answer choices. I originally couldn't see why D was correct, but upon further reflection thanks to JY's explanation I now do.

User Avatar
ryanrt579
Wednesday, Dec 16 2020

Great explanation Nabitou!

I got this question wrong. After reading your explanation, I can see that another way to look at Judy's response is that, they are stating that 80% of drivers do all of their driving within 5 miles and thus they don't do long distance driving, so that's something else that needs to be explained.

User Avatar
ryanrt579
Saturday, May 15 2021

Congratulations on your success scottshimi!

User Avatar
ryanrt579
Monday, Feb 15 2021

Well said FutureLawyer77, I've been studying full-time now for more than a year and I completely agree; your score doesn't define you: its just a snapshot of a moment in time. Improvement is possible and probable. My score has gone up, just like countless others. Great job keeping at it, I'm sure with your positive attitude and tenacity, that you will achieve the goal that you need!

PrepTests ·
PT121.S4.Q18
User Avatar
ryanrt579
Friday, Jan 15 2021

What messed me up about this question is that there are two completely separate conditional statements: Leadership and Following the Leader are NOT the same thing. Its important to recognize the nuances between words.

PrepTests ·
PT133.S1.Q7
User Avatar
ryanrt579
Wednesday, Apr 14 2021

What I found that helped me with this question was utilizing analogies to help understand the correct vs incorrect answer choices:

A) Virus A has these specific symptoms; Virus B does not- yes a weakener

B) Some people who have the virus, do not have these symptoms (we already know that since its not listed as everyone has the stimulus (i.e. "many") - Not a weakener

C) Yes its a weakner if the vector(host) animals for virus A are not in the location of the outbreak

D) Virus A infects 1 in 5; Virus B affects 1 in 1,000. Its hard to believe that the virus that only infected 1 in 1,000 could be Virus A. - weakener

E) This is the same rationale as D: if virus A lasts 2 months for the infection and Virus B lasts 1 year, then its likely that the virus that lasted for nearly one year in country X, is virus A.

Putting it in your own words (the answer choices and stimulus) can really help you see what's right and what's not.

User Avatar
ryanrt579
Monday, Dec 14 2020

Hello,

This question took me a while to understand why A was not the answer. Anson concludes that Dr. Ladlow isn't a responsible psychologist. The question stem asks: "Anson bases his conclusion about Dr. Ladlow based on which of the following?"

A ) If anything, the attack on his character would be the conclusion, not the support for the conclusion. Furthermore, from what I understand a personal attack would be more along the lines of: "Dr. Ladlow smokes cigarettes; we shouldn't believe anything he says"; not a professional criticism.

B ) Is correct because it takes the general principle within the stimulus of that responsible psychologists need to consider the potential of evidence that could refute their own findings, which Dr. Ladlow fails to do. Thus by failing to adhere to a general principle, Anson states that Dr. Ladlow's incorrect.

C ) There's no ambiguous term within this stimulus.

D ) Anson doesn't dispute Dr. Ladlow's facts (i.e. that the Dr.'s theory about rats isn't correct, its just that Anson adds to the notion that he must also consider the possibly that it might NOT be correct)

E ) Anson doesn't reject the Dr.'s theoretical explanation.

User Avatar

Monday, Dec 14 2020

ryanrt579

PT4.S4.Q18 - Dr. Ladlow's Theory

Hello,

This question took me a while to understand why A was not the answer. Anson concludes that Dr. Ladlow isn't a responsible psychologist. The question stem asks: "Anson bases his conclusion about Dr. Ladlow based on which of the following?"

A ) If anything, the attack on his character would be the conclusion, not the support for the conclusion. Furthermore, from what I understand a personal attack would be more along the lines of: "Dr. Ladlow smokes cigarettes; we shouldn't believe anything he says"; not a professional criticism.

B ) Is correct because it takes the general principle within the stimulus of that responsible psychologists need to consider the potential of evidence that could refute their own findings, which Dr. Ladlow fails to do. Thus by failing to adhere to a general principle, Anson states that Dr. Ladlow's incorrect.

C ) There's no ambiguous term within this stimulus.

D ) Anson doesn't dispute Dr. Ladlow's facts (i.e. that the Dr.'s theory about rats isn't correct, its just that Anson adds to the notion that he must also consider the possibly that it might NOT be correct)

E ) Anson doesn't reject the Dr.'s theoretical explanation.

User Avatar
ryanrt579
Tuesday, May 11 2021

Excellent and inspirational post, thank you!

User Avatar
ryanrt579
Thursday, May 06 2021

Congratulations Jeff!

User Avatar
ryanrt579
Wednesday, Feb 03 2021

Congratulations Nickes29! All the best!

PrepTests ·
PT132.S4.Q5
User Avatar
ryanrt579
Sunday, May 02 2021

Note: so in actuality, E is wrong because of it’s a classic trap of “belief vs fact”, flaw. Just because I believe that I defeated LeBron James in basketball, doesn’t mean that I actually did. B is correct because of an alternative cause. Which is the time-frame explanation. Its delayed: so everyone who is getting skin cancer now, has nothing to do with utilization of sunscreen, instead its just due to a delayed effect.

PrepTests ·
PT132.S4.Q7
User Avatar
ryanrt579
Sunday, May 02 2021

- During the course of three decades, researchers kept a record of average beak sizes between wild and captive birds

- During this period, they notice the size of the former decreased significantly and the latter stayed the same

Resolve the Discrepancy

A) This is tricky, but just because someone is easier to capture and measure, doesn’t mean they were successful in doing so. There’s also no time period listed, eliminate.

B) No, this wouldn’t explain it, if anything it should be larger then, eliminate

C) This is tricky; but its definitive in the sense that during the study period, the changes favoured the survival of the wild birds’ group, so this would lean in the area of resolving the issue. Also, if they didn’t survive, then there’s no way that they would be able to be measured.

D) This would confound it even further

E) Doesn’t interact, so what if they measured it on more than one occasion?

Correct answer: C

Confirm action

Are you sure?